Derivative of this piecewise function?

Click For Summary
The discussion centers on whether the derivative of a piecewise function should also be piecewise defined or can be expressed as a single continuous function. The piecewise function in question has defined intervals with no gaps, but it contains discontinuities. It is noted that the derivative will have even more discontinuities than the original function, suggesting that it cannot be simply written as a normal function. Graphing the function is recommended to visualize these properties. Ultimately, the conclusion is that the derivative should indeed be piecewise defined due to the nature of the original function's discontinuities.
Inertigratus
Messages
123
Reaction score
0

Homework Statement


I have this piecewise defined function and am wondering if the derivative is supposed to be piecewise defined as well?
Or could I just write it out on a line like a "normal" function?

Homework Equations


x(t) = {-14t : 0\leq|t|< 1, 2sgn(t) : 1\leq|t|< 7, (-12/7)sgn(t) : 7\leq|t|< 14, 0 : 14\leq|t| }

The Attempt at a Solution


I'm thinking I can write the derivative as a normal function. Since the piecewise functions are defined with no time-jumps between them and the derivative will have Dirac delta functions defined at the function incontinuities. Therefor the derivative should be continuous on the interval:
0\leq|t|\leq14.
Is this correct?
 
Physics news on Phys.org
Graph the function.

That should tell you the answer.

There are no gaps in the function definition -- if you are referring to the fact that the domain of the function is (-∞, ∞). However, this function has discontinuities at several places. Its derivative has even more discontinuities than the function itself !
 
Oh right, I asked a little too quick before thinking it through enough.
Thanks!
 
Question: A clock's minute hand has length 4 and its hour hand has length 3. What is the distance between the tips at the moment when it is increasing most rapidly?(Putnam Exam Question) Answer: Making assumption that both the hands moves at constant angular velocities, the answer is ## \sqrt{7} .## But don't you think this assumption is somewhat doubtful and wrong?

Similar threads

  • · Replies 5 ·
Replies
5
Views
2K
  • · Replies 4 ·
Replies
4
Views
2K
  • · Replies 7 ·
Replies
7
Views
3K
  • · Replies 7 ·
Replies
7
Views
1K
  • · Replies 22 ·
Replies
22
Views
3K
  • · Replies 20 ·
Replies
20
Views
3K
Replies
4
Views
4K
  • · Replies 3 ·
Replies
3
Views
2K
Replies
3
Views
2K
  • · Replies 40 ·
2
Replies
40
Views
4K